Question

In: Economics

Suppose there are two firms in a market who each simultaneously choose a quantity. Firm 1’s...

Suppose there are two firms in a market who each simultaneously choose a quantity. Firm 1’s quantity is q1, and firm 2’s quantity is q2. Therefore the market quantity is Q = q1 + q2. The market demand curve is given by P = 225 - 3Q. Also, each firm has constant marginal cost equal to 9. There are no fixed costs.

The marginal revenue of the two firms are given by:

  • MR1 = 225 – 6q1 – 3q2
  • MR2 = 225 – 3q1 – 6q2.

A) How much output will each firm produce in the Cournot equilibrium?

B) What will be the market price of the good?

C) What is the deadweight loss that results from this duopoly?

D) How much profit does each firm make?

E) Suppose Firm 2 produced 40 units of output. How much output should Firm 1 produce in order to maximize profit?

Solutions

Expert Solution

P = 225 - 3Q = 225 - 3q1 - 3q2

(A)

For firm 1, equating MR1 and MC,

225 - 6q1 - 3q2 = 9

6q1 + 3q2 = 216

2q1 + q2 = 72..........(1) (Best response, firm 1)

For firm 2, equating MR2 and MC,

225 - 3q1 - 6q2 = 9

3q1 + 6q2 = 216

q1 + 2q2 = 72..........(2) (Best response, firm 2)

Cournot equilibrium is obtained by solving (1) and (2). Multiplying (2) by 2,

2q1 + 4q2 = 144........(3)

2q1 + q2 = 72..........(1)

(3) - (1) yields:

3q2 = 72

q2 = 24

q1 = 72 - 2q2 [From (2)] = 72 - (2 x 24) = 72 - 48 = 24

(B)

Q = q1 + q2 = 24 + 24 = 48

P = 225 - (3 x 48) = 225 - 144 = 81

(C)

In social optimal output, P = MC.

225 - 3Q = 9

3Q = 216

Q = 72

P = MC = 9

Deadweight loss = (1/2) x Change in P x Change in Q = (1/2) x (81 - 9) x (72 - 48) = (1/2) x 72 x 24 = 864

(D)

Profit, Firm 1 = Q1 x (P - MC) = 24 x (81 - 9) = 24 x 72 = 1728

Profit, Firm 2 = Q2 x (P - MC) = 24 x (81 - 9) = 24 x 72 = 1728

NOTE: As per Answering Policy, 1st 4 parts are answered.


Related Solutions

Suppose there are two firms in a market who each simultaneously choose a quantity. Firm 1’s...
Suppose there are two firms in a market who each simultaneously choose a quantity. Firm 1’s quantity is q1, and firm 2’s quantity is q2. Therefore the market quantity is Q = q1 + q2. The market demand curve is given by P = 100 – 2Q. Also, each firm has constant marginal cost equal to 10. There are no fixed costs. The marginal revenue of the two firms are given by: MR1 = 100 – 4q1 – 2q2 MR2...
Suppose there are two firms in a market who each simultaneously choose a quantity. Firm 1’s...
Suppose there are two firms in a market who each simultaneously choose a quantity. Firm 1’s quantity is q1, and firm 2’s quantity is q2. Therefore the market quantity is Q = q1 + q2. The market demand curve is given by P = 160 - 2Q. Also, each firm has constant marginal cost equal to 10. There are no fixed costs. The marginal revenue of the two firms are given by: MR1 = 160 – 4q1 – 2q2 MR2...
Suppose there are two firms in a market who each simultaneously choose a quantity. Firm 1’s quantity is q1, and firm 2’s quantity is q2.
Suppose there are two firms in a market who each simultaneously choose a quantity. Firm 1’s quantity is q1, and firm 2’s quantity is q2. Therefore the market quantity is Q = q1 + q2. The market demand curve is given by P = 160 - 2Q. Also, each firm has constant marginal cost equal to 10. There are no fixed costs.The marginal revenue of the two firms are given by:MR1 = 160 – 4q1 – 2q2MR2 = 160 –...
Suppose there are two firms in a market who each simultaneously choose a quantity.Firm 1's quantity...
Suppose there are two firms in a market who each simultaneously choose a quantity.Firm 1's quantity is q1, and firm 2's quantity is q2. Therefore the market quantity is Q= q1+q2. The market demand curve is given by P=130-Q. Also, each firm has constant marginal cost equal to 25. There are no fixed costs. Marginal revenue of the firms are given by MR1=130-2q1-q2 & MR2=130-q1-2q2. A) How much output will each firm produce in the Cournot equilibrium ? B) What...
Suppose there are two firms in a market who each simultaneouslychoose a quantity. Firm 1’s...
Suppose there are two firms in a market who each simultaneously choose a quantity. Firm 1’s quantity is q1, and firm 2’s quantity is q2. Therefore the market quantity is Q = q1 + q2. The market demand curve is given by P = 100 – 4Q. Also, each firm has constant marginal cost equal to 28. There are no fixed costs.The marginal revenue of the two firms are given by:MR1 = 100 – 8q1 – 4q2MR2 = 100 –...
two firms produce a homogenous good, each firm simultaneously decides on its quantity to produce and...
two firms produce a homogenous good, each firm simultaneously decides on its quantity to produce and respectively letting the market determine the price. the market price for both firms is   P= 4020-10(q1+q2). firm 1 faces marginal cost c1=40. firm faces marginal cost c2=80. 11.1 write down the profit function for each firm as a function of q1 and q2 .2 Determine the best response function of each firm and draw a graph of the best response curves .3 determine the quantities...
Two firms, Firm 1 and Firm 2, compete by simultaneously choosing prices. Both firms sell an...
Two firms, Firm 1 and Firm 2, compete by simultaneously choosing prices. Both firms sell an identical product for which each of 100 consumers has a maximum willingness to pay of $40. Each consumer will buy at most 1 unit, and will buy it from whichever firm charges the lowest price. If both firms set the same price, they share the market equally. Costs are given by ??(??)=16??ci(qi)=16qi. Because of government regulation, firms can only choose prices which are integer...
Suppose there are only two firms in the market, firm 1 and firm 2. They produce...
Suppose there are only two firms in the market, firm 1 and firm 2. They produce identical products. Firm 1 has a constant marginal cost where AC1 =MC1 =20, and firm 2 has a constant marginal cost AC2 =MC2 =8. The market demand function is given by Q = 100 - 0.5P. a) Find the Cournot Nash Equilibrium price and quantity, write down the profits for each firm. (Use "q1" to represent output level for firm 1, and "profit1" to...
Consider a Cournot-competition under incomplete information. Two firms decide their quantity of production simultaneously. The market...
Consider a Cournot-competition under incomplete information. Two firms decide their quantity of production simultaneously. The market price P is determined by P = 100 − (q1 + q2). Assume that firm 1’s per-unit cost is commonly known at zero. On the other hand, firm 2’s per-unit cost is private information and is either at 0 or at 2. Suppose in the firm 1’s belief, the probability of c2 = 0 is 1 3 and the probability of c2 = 2...
Consider Cournot model of quantity competition between two firms, firm 1 and firm 2. Suppose the...
Consider Cournot model of quantity competition between two firms, firm 1 and firm 2. Suppose the inverse demand for the firms product is given by ?=40(?+2)−(?+1)(?1+?2)p=40(A+2)−(B+1)(q1+q2), where ??qi denotes the quantity of firm ?i, ?=1,2i=1,2, ?A is 6 and ?B is 9. Each firm's average cost is equal to ?+2c+2, where ?c is 4. a) Derive and accurately plot each firm's best response function. b) Find the (Nash) equilibrium quantities, price, profits and consumer surplus. c) Suppose next that each...
ADVERTISEMENT
ADVERTISEMENT
ADVERTISEMENT